Answer (B) is correct . Nerney’s maximum net profit can be calculated as follows: ? Selling price $?????????12 Less:? variable costs (8) Contribution margin per unit $???????????4 Times:? unit sales ×??40,000 Total contribution margin $160,000 Less:? fixed costs (48,000) Operating income $112,000 Less:? income taxes (40%) (44,800) Net profit $??67,200
Answer (A) is incorrect because The amount of fixed costs is $48,000. Answer (C) is incorrect because The amount of $96,000 results from failing to subtract the fixed costs. Answer (D) is incorrect because The operating profit before taxes is $112,000.
|